Intégrale sous contrainte

On peut donner une forme close* de l'intégrale $\displaystyle \int_0^1 \dfrac{\ln x\ln(1+x)}{1+x}dx$ en utilisant seulement comme outils : intégration par parties et changement de variable dans des intégrales avec une seule variable (et sans utilisation de fonctions spéciales).

Je me demandais si on pouvait faire un calcul similaire avec :
$\displaystyle \int_0^1 \dfrac{\ln x\ln(1-x)}{1+x}dx$**
(je n'en ai pas l'impression).

*. Bien sûr, pour ce faire, on suppose connue une expression de l'intégrale $\displaystyle \int_0^1 \dfrac{\ln^2 x}{1-x}dx$
Et il ne faut pas compter sur Wolfy pour indiquer ce qu'il faut faire pour y arriver. B-)-
On peut montrer sans recours à des fonctions de deux variables, ou fonctions spéciales que $\displaystyle \int_0^1 \dfrac{\ln x\ln(1+x)}{1+x}dx=\frac{1}{16} \int_0^1 \dfrac{\ln^2 x}{1-x}dx$

**. C'est-à-dire montrer que $\displaystyle \int_0^1 \dfrac{\ln x\ln(1-x)}{1+x}dx=\frac{13}{16} \int_0^1 \dfrac{\ln^2 x}{1-x}dx+\frac{3}{2}\ln 2 \int_0^1 \dfrac{\ln x}{1-x}dx.$

Réponses

  • Bonjour FDP, y a-t-il une récompense si qulqu'un trouve? Je sais que ta cave regorge de pleins anciens livres. Tu mets quoi dans ce prix comme livres à gagner?
    Ajout, Pour comprendre mes propos, ce n'est pas la première fois que tu poses le même calcul de cette intégrale avec des méthodes restrictives.
    Le 😄 Farceur


  • Bonjour,

    On calcule, avec $\displaystyle I=[0,1]$, $\displaystyle \int_I {\ln^2 (x(1-x)) \over 1+x} dx= \int_I {\ln^2 x + \ln^2 (1-x) + 2 \ln x \ln(1-x) \over 1+x} dx.$

    Par simple dérivation, on vérifie :
    qu'une primitive de $\displaystyle x \mapsto {\ln^2 x \over 1+x} $ est la fonction $\displaystyle x \mapsto -2 Li_3(-x) + 2 Li_2(-x) \ln x + \ln^2 x \ln(1+x)$,
    qu'une primitive de $\displaystyle x \mapsto {\ln^2 (1-x) \over 1+x} $ est la fonction $\displaystyle x \mapsto -2 Li_3({1 - x \over 2}) + 2 Li_2({1 - x \over 2}) \ln (1-x) + \ln^2 (1-x) \ln({1 + x \over 2})$,
    qu'une primitive de $\displaystyle x \mapsto {\ln^2 (x(1-x)) \over 1+x} $ est la fonction https://www.wolframalpha.com/input/?i=integrate+ln(x/(1-x))^2/(1+x)+

    Et on calcule le tout en $0$ et en $1$ aux limites.

    Voilà !
  • Bonsoir, moi je vois poindre la-dedans le dilogarithme et des intégrales doubles mais bon...
    A demon  wind propelled me east of the sun
  • YvesM: Mais tu ne respectes pas la contrainte que j'ai mentionnée.

    Comme déjà indiqué il y a un truc curieux qui se produit selon moi.

    On peut calculer* $\displaystyle \int_0^1 \dfrac{\ln x\ln(1+x)}{1+x}dx$ sans dilogarithme, sans intégrale double.
    Mais, à ma connaissance de cet instant, on ne peut pas calculer $\displaystyle \int_0^1 \dfrac{\ln x\ln(1-x)}{1+x}dx$ de la même façon aussi élémentaire.

    *: j'ai écris plus haut ce que j'entendais par calculer: trouver une relation simple entre $\displaystyle \int_0^1 \dfrac{\ln x\ln(1+x)}{1+x}dx$ et $\displaystyle \int_0^1 \dfrac{\ln^2 x}{1-x}dx$
  • \begin{align}K&=\int_0^1\frac{\ln(1+x)\ln x}{1+x}dx\\&=\frac{1}{2}\left(\int_0^1\frac{\ln^2(1+x)}{1+x}dx+\int_0^1\frac{\ln^2 x}{1+x}dx-\underbrace{\int_0^1 \frac{\ln^2\left(\frac{x}{1+x}\right)}{1+x}dx}_{y=\frac{x}{1+x}}\right)\\
    &=\frac{1}{6}\ln^3 2+\frac{1}{2}\int_0^1\frac{\ln^2 x}{1+x}dx-\frac{1}{2}\int_0^{\frac{1}{2}}\frac{\ln^2 y}{1-y}dy\\
    \end{align}
    Par ailleurs,
    \begin{align}
    K&\overset{y=\frac{1}{1+x}}=-\int_{\frac{1}{2}}^1\frac{\ln\left(\frac{1-y}{y}\right)\ln y}{y}dy\\
    &=\frac{1}{3}\ln^3 2-\int_{\frac{1}{2}}^1 \frac{\ln\left(1-y\right)\ln y}{y}dy\\
    &\overset{\text{IPP}}=\frac{1}{3}\ln^3 2-\frac{1}{2}\Big[\ln^2 y\ln(1-y)\Big]_0^1 +\frac{1}{2}\int_{\frac{1}{2}}^1 \frac{\ln^2 y}{1-y}dy\\
    &=-\frac{1}{6}\ln^3 2-\frac{1}{2}\int_{\frac{1}{2}}^1 \frac{\ln^2 y}{1-y}dy\\
    &=-\frac{1}{6}\ln^3 2-\frac{1}{2}\int_0^1 \frac{\ln^2 y}{1-y}dy+\frac{1}{2}\int_0^{\frac{1}{2}} \frac{\ln^2 y}{1-y}dy
    \end{align}
    Donc:
    \begin{align}2K&=-\frac{1}{2}\int_0^1 \frac{2x\ln^2 x}{1-x^2}dx\\
    &\overset{y=x^2}=-\frac{1}{8}\int_0^1 \frac{\ln^2 y}{1-y}dy\\
    K&=\boxed{-\dfrac{1}{16}\int_0^1 \dfrac{\ln^2 y}{1-y}dy}\\
    \end{align}

    Comme déjà indiqué, j'ai des doutes sur le fait qu'on puisse dérouler un calcul analogue avec l'intégrale soeur $\displaystyle \int_0^1 \dfrac{\ln(1-x)\ln x}{1+x}dx$
  • Pour toute fin utile
    $$\int_0^1 \frac{\ln (1-x)\ln x}{1+x} dx =\frac{13}8\zeta(3) -\frac{\pi^2\ln(4) }{8}$$
    Le 😄 Farceur


  • Un calcul de l'intégrale $\displaystyle J= \int_0^1\frac{\ln x\ln(1-x)}{1+x}dx$ mais qui, hélas, ne respecte pas la contrainte demandée.

    \begin{align}
    J&\overset{\text{IPP}}=\left[\left(\int_0^x \frac{\ln t }{1+t}dt -\int_0^1 \frac{\ln t }{1+t}dt\right)\ln(1-x)\right]_0^1+\int_0^1 \frac{1}{1-x}\left(\int_0^x \frac{\ln t }{1+t}dt-\int_0^1 \frac{\ln t }{1+t}dt\right)dx\\
    &=\int_0^1 \int_0^1 \frac{1}{1-x}\left(\frac{x\ln(tx)}{1+tx}-\frac{\ln t}{1+t}\right)dtdx\\
    &=\int_0^1 \int_0^1\left(\frac{\ln(tx)}{(1+t)(1-x)}-\frac{\ln(tx)}{(1+t)(1+tx)}-\frac{\ln t}{(1+t)(1-x)}\right)dtdx\\
    &=-\frac{1}{6}\pi^2\ln 2-\int_0^1 \frac{1}{t(1+t)} \left(\int_0^t\frac{\ln u}{1+u}du\right)dt\\
    &\overset{\text{IPP}}=-\frac{1}{6}\pi^2\ln 2+\left[\big(\ln(1+t)-\ln t \big)\left(\int_0^t\frac{\ln u}{1+u}du\right)\right]_0^1+\int_0^1\frac{\ln^2 t}{1+t}dt-\int_0^1\frac{\ln(1+t)\ln t}{1+t}dt\\
    &=\frac{3}{2}\zeta(3)-\frac{1}{4}\pi^2\ln 2-\int_0^1\frac{\ln(1+t)\ln t}{1+t}dt\\
    \end{align}
    Et, avec le calcul fait dans le message précédent, on a donc:
    $\boxed{\displaystyle J=\frac{13}{8}\zeta(3)-\frac{1}{4}\pi^2\ln 2}$

    NB: J'admets que,
    \begin{align}\int_0^1\frac{\ln t}{1+t}dt&=-\frac{1}{12}\pi^2,
    \int_0^1\frac{\ln t}{1-t}dt=-\frac{1}{6}\pi^2,
    \int_0^1\frac{\ln^2 t}{1+t}dt=\frac{3}{2}\zeta(3),
    \int_0^1\frac{\ln^2 t}{1-t}dt=2\zeta(3)\\
    \end{align}
Connectez-vous ou Inscrivez-vous pour répondre.